LSAT and Law School Admissions Forum

Get expert LSAT preparation and law school admissions advice from PowerScore Test Preparation.

 Adam Tyson
PowerScore Staff
  • PowerScore Staff
  • Posts: 5153
  • Joined: Apr 14, 2011
|
#74640
Complete Question Explanation

Flaw in the Reasoning. The correct answer choice is (A).

The author tells us that it is possible, albeit expensive, to build moon colonies, and that the population is growing and we are running out of space on Earth. This gives us an incentive to build those colonies, and the author then concludes that we will therefore almost certainly build them someday. A good abstraction of this argument would be "We can do X, and there is a reason to do X, so we will probably do X."

When put that way, the flaw should be clear: being able to do something and having a reason to do it does not necessarily mean that we will actually do it. Perhaps there will always be more reasons NOT to do it than to do it? Look for an answer that points out that the author has failed to consider the relative strength of the reasons for building moon colonies vs the strength of the reasons for not building them.

Answer choice (A): This is the correct answer choice. This answer captures the author's failure to consider the relative strength of the reasons for and against building moon colonies. Even with the incentive to build them growing, it could be that the reasons against building them will always outweigh the reasons for doing so.

Answer choice (B): The argument does NOT assume that building moon colonies will be the only solution to the overcrowding problem, but only that the reasons for doing so will be more persuasive than the reasons for not doing so. Correct flaw answers must describe something that actually happened in the argument, and the author never went to such an extreme as to imply this was the only option. One further argument against this potentially attractive answer is found in the conclusion, where the author said "almost certainly" rather than "certainly."

Answer choice (C): This answer describes something that did NOT happen in the stimulus! The author did not toverlook the possibility that moon colonies would be built - the author said that they would be!

Answer choice (D): While the argument never addressed the possibility described in this answer choice, and so it could be said that they overlooked this, this would not constitute a flaw of the argument because the author is only claiming that it is likely that such colonies will be built, and not that doing so would be a complete and perfect solution to the overcrowding problem.

Answer choice (E): The argument has nothing to do with what humans would prefer, and it would not be a problem for this author if some people would like to live on the moon even if Earth was not overcrowded. The author did not take such a possibility for granted because it would not matter either way.
 rachue
  • Posts: 140
  • Joined: Jun 22, 2011
|
#1126
In this question, I thought b was the best answer because it seemed more probable in the real world. Can someone please explain why A is the correct answer?

Question removed by Admin. The text of LSAT questions cannot be posted on the board.
User avatar
 Dave Killoran
PowerScore Staff
  • PowerScore Staff
  • Posts: 5853
  • Joined: Mar 25, 2011
|
#1157
The problem with answer choice (B) is the word "only." There's nothing in Max's argument suggesting that he assumes this is the only way of relieving overcrowding; instead, he's just addressing the possibilities of this option.

Answer choice (A) accurately describes a flaw in Max's argument. Max says that because we already possess the technology, despite the project being so costly there will be a "growing economic incentive," and so colonies will "almost certainly" get built. Think about that for a moment: it's extremely expensive, but possible, so growing incentives dictate that they will almost surely get built. That line of reasoning assumes that the economic incentives will be so great as to justify taking on the costly project (which is what A states). Is that a reasonable assumption? I don't think so. From a commonsense standpoint, building anything on the moon would be astronomically expensive, and there are likely other, cheaper ways to provide housing on earth before incurring the same costs. Thus, I don't think we can say that housing on the moon will almost certainly get built. A describes the assumption as flawed, and thus A is the correct answer.

Let me know if that makes sense. Thanks!
 rachue
  • Posts: 140
  • Joined: Jun 22, 2011
|
#1159
That does make sense. Thanks!
 SLF
  • Posts: 40
  • Joined: Oct 01, 2013
|
#13032
With regard to LSAT #54, Section #4, Question #16, how should I discriminate between answer choices 'A' and 'B'?
 Jacques Lamothe
PowerScore Staff
  • PowerScore Staff
  • Posts: 50
  • Joined: Sep 24, 2013
|
#13036
Hey SLF,

The stimulus argument claims that the increasing human population increases the economic inceptive to build Moon colonies. The author's conclusion seems to assume that Moon colonies will be built because of these increasing icnentives. But the author mentions that those colonies would be very expensive to build. So increasing economic incentives do not guarantee that the colonies will be built. Those new incentives would have to be large enough that someone would pay the building costs. Since this is something that the author does not account for in the stimulus, answer choice (A) would be a correct criticism of the argument.

Answer choice (B) ends up being incorrect because the stimulus never argues or relies on an assumption that Moon colonies are the only way to relieve overcrowding. The author only argues that Moon colonies will be built, not that other ways of relieving overcrowding will not occur as well.

I hope this helps!

Jordan
 karen_k
  • Posts: 35
  • Joined: Sep 24, 2015
|
#19978
Hi,

Can you please explain why B is wrong and therefore why A is better than B? Thank you.
 David Boyle
PowerScore Staff
  • PowerScore Staff
  • Posts: 836
  • Joined: Jun 07, 2013
|
#20000
karen_k wrote:Hi,

Can you please explain why B is wrong and therefore why A is better than B? Thank you.
Hello karen_k,

B is not the worst answer, though the stimulus never says that there is no other way to deal with the overcrowding. Also, the stimulus says, "such colonies will almost certainly be built", instead of "certainly", leaving open the possibility that some other avenue of relief exists.
A deals more directly with the stimulus, that is, the economic incentive is assumed by the stimulus (maybe falsely assumed, as answer A implies) to be strong enough to overcome objections based on cost of the project.

Hope this helps,
David
 karen_k
  • Posts: 35
  • Joined: Sep 24, 2015
|
#20024
Thank you, David!
 Sherry001
  • Posts: 81
  • Joined: Aug 18, 2014
|
#20221
Hi there ,
Could you please explain why C is wrong?

1- though costly , humans have the tech
2- human population may increase and the amount of space for housing decreases
3- increasing economic incentive to construct such colonies

C: we will almost certainly built such colonies .

My analysis before looking at the answer choices was something like this : well just because we have incentive doesn't mean we will for sure build housing on moon. What if the costs are more then the economic incentive ? What if govnemnt begins to control increasing population so we stay in line with housing demands ?

A) I see why this is the correct answer.

B) author never says only Anything.

C) this choice seemed kinda attractive . What if it's not because of the incentive but rather quality of life that we may want to move to moon ?

D) so what this has nothing to do with building life there.

e) irrelevant .

Get the most out of your LSAT Prep Plus subscription.

Analyze and track your performance with our Testing and Analytics Package.